show work help plsss

Show Work Help Plsss

Answers

Answer 1

The required quotient is 6.5625 when -5 2/8 is divided by (-4)/5.

What is the Quotient?

A quotient is defined as the outcome of dividing an integer by any divisor that can be said to be a quotient. The dividend contains the divisor a specific number of times.

The quotient is given in the question:

-5 2/8 ÷ (-4)/5

-(42/8) / (-4)/5

Reciprocal the term in the denominator

-(42/8) × -(5)/4

210/32

6.5625

Therefore, the required quotient is -5 2/8 ÷ (-4)/5 = 6.5625.

Learn more about the quotient here:

brainly.com/question/27796160

#SPJ1


Related Questions

A constant force is applied to an object, causing the object to accelerate at 12.50 m/s/s. What will the acceleration be if both the force and the
mass are doubled? Round to two decimal places and use SI units.

Answers

It doesn’t give the other shape the measurements. What grade level are you?

Answer:

a₂ = 12.5 m/s²

Step-by-step explanation:

Applying the 2º Newton's Law:

F = m * a

F₁ = m₁ * a₁

F₁ = m₁ * 12.5

F₁ = 12.5m₁

F₂ = m₂ * a₂

2F₁ = 2m₁ * a₂

a₂ = F₁ / m₁

So:

a₂ = 12.5m₁ / m₁

a₂ = 12.5 m/s²

Describe the transformation that maps the pre-image A to the image A.translated 8 units up and then reflected across the y-axisB)translated 8 units down and then reflected across the y-axisC)translated 1 unit to left and then rotated 180° counterclockwise about theoriginD)translated 1 unit to right and then rotated 180° counterclockwise about theorigin

Answers

Answer:

C. translated 1 unit to left and then rotated 180° counterclockwise about the

origin

Explanation:

In the graph:

• The vertices of Figure A are: (0,4), (-9,3) and (-9,6)

,

• The vertices of Figure A' are: (1,-4), (10,-3) and (10,-6)

If Figure A is translated 1 unit to the left, the vertices are:

[tex](-1,4),(-10,3),(-10,6)[/tex]

Next, a rotation of 180° counterclockwise about the origin gives:

[tex](1,-4),(10,-3),(10,-6)[/tex]

These are the vertices of Figure A'.

Thus, the correct transformation is: translated 1 unit to left and then rotated 180° counterclockwise about the origin.

The correct choice is C.

Kendall read 1 1/3 books in a 1/2day. At that rate how many books will she read in
1 day?

Answers

Answer:

Step-by-step explanation:

1/2 to 1 day is 2 1/2

11/3 x 11/3 = 121/9

13.4 books

Two numbers are respectively 60% and 20% of a third number, second number exepress ss as a perentage of first is?​

Answers

Answer:

10%

Step-by-step explanation:

this is because the total number is supposed to be 100%

and 60%+20%=90%. 100%-90%=10%.

So 10% is the final answer

25×10 to the power of -3 to standard form

Answers

The number 25×10 to the power of -3 written in the standard form is 2.5×10⁻⁴.

What is meant by the term scientific notations?Scientific notation is a way of displaying very large or extremely small numbers in a more simplified format. We know that whole numbers could be extended indefinitely, but such large numbers cannot be written on a sheet of paper. The scientific notation allows us to represent very large or very small numbers by multiplying single-digit numbers by 10 raised to a power of a respective exponent. If the number is so very large, the exponent is positive; otherwise, it is negative. For a better understanding, learn power and exponents.

The given number is;

25×10 to the power of -3

This can be written as;

= 25×10⁻³

Now, as per the standard form, the number should contain only one digit to the left of decimal.

Thus, shifting one decimal to the left of the 5 and converting the power by adding one to the current power,.

The number becomes;

= 2.5×10⁻⁴.

Thus, the number written in the standard form is 2.5×10⁻⁴.

To know more about the scientific notations, here

https://brainly.com/question/1767229

#SPJ10

Jason made a spherical cake pop that is 1.3 inches in diameter. He coated the whole cake pop with sprinkles.What is the approximate area Jason covered with sprinkles?

Answers

Given:

The diameter of the sphere is 1.3 inches.

The surface area of the sphere is given as,

[tex]\begin{gathered} SA=4\pi\times r^2 \\ SA=4\pi\times(\frac{d}{2})^2 \\ SA=4\pi\times(\frac{1.3}{2})^2 \\ SA=1.69\pi \\ SA=1.69\times3.14\ldots\ldots\pi=3.14 \\ SA=5.3066 \\ SA\approx5.3\text{ square inches} \end{gathered}[/tex]

Answer: area covered with sprinkles is 5.3 square inches

A passenger car will go 310 miles on 15.5 gallons of gasoline in city driving. What is the rate in miles per gallon?
The rate is
mpg.
(Simplify your answer.)

Answers

Answer:

20 mpg

Step-by-step explanation:

Given:

A passenger car will go 310 miles on 15.5 gallons of gasoline in city driving. What is the rate in miles per gallon?

Solve:

To find the rate in miles per gallon we have to divide.

Knowing that "Per" means "Each/One"

Hence, to find miles per gallon we divide :

Miles by Gallons

⇒ 310/15.5

Solving using long division format: ( Multiply both by 10 first)

[tex]155\overline{|\smallspace3100}\:\:\:\:[/tex]

Divide 310 by 155 to get 2:

[tex]\begin{matrix}\:\:\:\:\:\:\:\:\:\:\:\:\emptyspace2\:\:\:\:\:\:\:\:\:\:\:\:\:\:\\ 155\overline{|\smallspace3100}\:\:\:\:\:\:\:\:\:\:\:\:\\ \:\:\:\:\:\:\:\:\underline{\emptyspace3\emptyspace1\emptyspace0}\:\:\:\:\:\:\:\:\:\:\:\:\:\:\\ \:\:\:\:\:\:\:\:\:\:\:\:\emptyspace0\emptyspace0\:\:\:\:\:\:\:\:\:\:\:\:\end{matrix}[/tex]

Divide 0 by 15.5 to get 0.

[tex]\begin{matrix}\:\:\:\:\:\:\:\:\:\:\:\:\emptyspace2\emptyspace0\:\:\:\:\:\:\:\:\:\:\:\:\\ 155\overline{|\smallspace3100}\:\:\:\:\:\:\:\:\:\:\:\:\\ \:\:\:\:\:\:\:\:\underline{\emptyspace3\emptyspace1\emptyspace0}\:\:\:\:\:\:\:\:\:\:\:\:\:\:\\ \:\:\:\:\:\:\:\:\:\:\:\:\emptyspace0\emptyspace0\:\:\:\:\:\:\:\:\:\:\:\:\\ \:\:\:\:\:\:\:\:\:\:\:\:\:\:\underline{\emptyspace0}\:\:\:\:\:\:\:\:\:\:\:\:\\ \:\:\:\:\:\:\:\:\:\:\:\:\:\:\emptyspace0\:\:\:\:\:\:\:\:\:\:\:\:\end{matrix}[/tex]

Hence, the rate is 20 mpg.

~kavinsky

please help im begging ill give brainliest

Answers

Answer:

t=44 hrs.

Step-by-step explanation:

1. a. you are asked to find the total hours the 2 girls work in a week.

b. it needs to be a greater number than 26

2. variables: h= hours per week

t= total hrs.

3. h= hrs. per week

t= total hrs.

4. 44=26+h

-26 -26

18=h

5. 44=26+18

44=44

6. The total hours the two work in a week is 44 hours.

The area of a circle is 100π cm². What is the circumference, in centimeters? Express your answer in terms of \piπ. THE ACTUAL ANSWER PLEASE

Answers

Answer:

35.45

Step-by-step explanation:

C=2πA=2·π·100≈35.44908

A large group is ordering shirts for a company outing. The order will be based on how many employees sign up for a shirt.

The independent variable in this situation is the size of the order.

True

OR

False

Answers

Answer:

False

Step-by-step explanation:

The dependent variable changes, the size of the order will change depending on how many employes want a shirt.

Which expression does not belong with the other three? justify your response.

Answers

To find the equation that does not belong with the other three we have to solve each equation.

Given the equation:

[tex]4\cdot x\cdot7=56[/tex]

Dividing by 4 at both sides:

[tex]\begin{gathered} \frac{4\cdot x\cdot7}{4}=\frac{56}{4} \\ x\cdot7=14 \end{gathered}[/tex]

7:

[tex]\begin{gathered} \frac{x\cdot7}{7}=\frac{14}{7} \\ x=2 \end{gathered}[/tex]

[tex]5\cdot2\cdot x=40[/tex]

Simplifying on the left:

[tex]10\cdot x=40[/tex]

10:

[tex]\begin{gathered} \frac{10\cdot x}{10}=\frac{40}{10} \\ x=4 \end{gathered}[/tex]

:

[tex]x\cdot3\cdot9=54[/tex]

[tex]x\cdot27=54[/tex]

27:

[tex]\begin{gathered} \frac{x\cdot27}{27}=\frac{54}{27} \\ x=2 \end{gathered}[/tex]

[tex]4\cdot x\cdot5=40[/tex]

4 :

[tex]\begin{gathered} \frac{4\cdot x\cdot5}{4}=\frac{40}{4} \\ x\cdot5=10 \end{gathered}[/tex]

5

[tex]\begin{gathered} \frac{x\cdot5}{5}=\frac{10}{5} \\ x=2 \end{gathered}[/tex]

In consequence, the equation that does not belong with the other three is:

[tex]5\cdot2\cdot x=40[/tex]

because its solution, 4, is different from the solution to the other equations (2).

If we change the 5 in the equation by a 10 and solve the equation we get:

[tex]\begin{gathered} 10\cdot2\cdot x=40 \\ \frac{10\cdot2\cdot x}{10}=\frac{40}{10} \\ 2\cdot x=4 \\ \frac{2\cdot x}{2}=\frac{4}{2} \\ x=2 \end{gathered}[/tex]

And now the equation (with a 10) belongs with the other three


28•x=56 1)x=2
10•x=40 2)x=4
27•x=54. 3)x=2
20•x=40. 4)x=2

good luck!

15. Over what range of angles does the value of sin(O) consistently increase?

A. 45° to 135°
B. 90° to 180°
C. 0° to 180°
D. 0° to 90°

Answers

The value of sin(O) will at the range of 0° to 90° consistently increase. This makes option D the correct options.

how to find the range of values where sin will consistently increase

In trigonometry, the first basic functions are the sin, cos and tan.

In a right angle triangle the sin is represented as the ratio of opposite to hypotenuse. cos is represented as the ratio of the adjacent to hypotenuse, while tan is the ratio of opposite to adjacent or ratio of sin to cos.

Plotting the graph of sin functions, the pattern forms a sinusoidal pattern which bears its maximum value at sin 90degrees and the minimum value at sin 0 degrees.

Read more on sin here: https://brainly.com/question/68324

#SPJ1

A spinner with 10 equally sized slices has 5 red slices , and 3 yellow slices, and 2 blue slices. The dial 500 times and got the following results. What is the probability that the dial stops on a red or yellow slice?

Answers

Part a

The experimental probability that landing on red is equal to

P=277/500

P=0.554

Part b

The theoretical probability that landing on red is equal to

P=5/10

P=0.5

Part c

the answer is the first option

Find the x-intercept of the line that passes through (-3,4) and is perpendicular to the graph of x-2y=7

Answers

Answer:

x - intercept = - 1

Step-by-step explanation:

the equation of a line in slope- intercept form is

y = mx + c ( m is the slope and c the y- intercept )

given

x - 2y = 7 ( subtract x from both sides )

- 2y = - x + 7 ( divide through by - 2 )

y = [tex]\frac{1}{2}[/tex] x + [tex]\frac{7}{2}[/tex] ← in slope- intercept form

with slope m = [tex]\frac{1}{2}[/tex]

given a line with slope m then the slope of a line perpendicular to it is

[tex]m_{perpendicular}[/tex] = - [tex]\frac{1}{m}[/tex] = - [tex]\frac{1}{\frac{1}{2} }[/tex] = - 2 , then

y = - 2x + c ← is the partial equation of the perpendicular line\to find c substitute (- 3, 4 ) into the partial equation

4 = 6 + c ⇒ c = 4 - 6 = - 2

y = - 2x - 2 ← equation of perpendicular line

to find the x- intercept let y = 0 into the equation and solve for x

- 2x - 2 = 0 ( add 2 to both sides )

- 2x = 2 ( divide both sides by - 2 )

x = - 1

the x- intercept of the perpendicular line is - 1 or (- 1, 0 )

solution a is 2% alcohol and solution b is 5% alcohol. she uses 800 milliliters of solution A how many milliliters of solution b does she use if the resulting mixture is a 3% alcohol situation

Answers

She uses 800 mL of solution A; that is, 2% of those 800 mL is alcohol

Alcohol on a:

2% of 800 = 2*800/100 = 16mL

If we use x mL of b, the amount of alcohol is:

5% of x = 5*x/100 = x / 20

Total amount of alcohol: 16 + x/20

Total amount of solution mixture: 800 + x

The ratio:

(16 + x/20) / (800 + x) = 3%

(16 + x/20) / (800 + x) = 3/100

1600 + 5x = 2400 + 3x

Solving for x:

5x - 3x = 2400 - 1600

2x = 800

x = 400 mL

She used 400 mL of solution b

Slope intercept form for points through (0,4) and (-3,3)

Answers

Answer:

y = (1/3)x + 4

Step-by-step explanation:

(0, 4) and (-3, 3)

(x₁, y₁)         (x₂, y₂)

        y₂ - y₁           3 - 4           -1            1

m = ------------- = ------------ = --------- = --------

         x₂ - x₁          -3 - 0         -3           3

y - y₁ = m(x - x₁)

y - 4 = (1/3)(x - 0)

y - 4 = (1/3)x - 0

   +4            +4

----------------------------

y = (1/3)x + 4

I hope this helps!

BenchenIf these two lines were graphed, where would they intersect? Drag the point to theirintersection's coordinates.y = x - 3y = -X- 1

Answers

y = x - 3

y = - x -1

They intersect at

how do I solve this
1/9 = z/23

Answers

Answer: z=23/9

Step-by-step explanation:

I will use cross multiplication

[tex]\frac{1}{9} =\frac{z}{23} \\23(1)=9(z)\\23=9z\\\frac{23}{9} =\frac{9z}{9} \\z=\frac{23}{9}[/tex]

I hope this helps.

the lewis family drove 45.6 miles to the river. It took them 1.2 hours to get there. How fast did they drive? Round to the nearest whole number

Answers

The most appropriate choice for speed will be given by

Speed of the car of Lewis family = 38 miles/hours

What is speed?

Distance travelled by an object per unit of time is called speed.

If d is the distance travelled and t is the time taken, then speed is calculated by-

Speed = [tex]\frac{d}{t}[/tex]

Here, speed is a scalar as the direction is not taken into account.

Distance drove by  Lewis family to the river = 45.6 miles

Time taken by the Lewis family to go to the river  = 1.2 hours

Speed of the car of Lewis family = [tex]\frac{45.6}{1.2}[/tex]

                                                      = 38 miles/hour

To learn more about speed, refer to the link-

https://brainly.com/question/26046491

#SPJ9

what does this mean 3|-3| the whole |-3| thing I'm confused please help ​

Answers

|-3| means "the absolute value of 3", which is a way of saying how far away from zero the number -3 is on the number line.

The definition of absolute value goes like this:

[tex]|x| = \begin{cases} x & \text{if } x \ge 0 \\ -x & \text{if } x < 0 \end{cases}[/tex]

In other words, if the number [tex]x[/tex] inside the absolute value bars is already positive or zero, we leave it alone. If instead [tex]x[/tex] is negative, we multiply it by -1 to recover the positive version of that number.

Since -3 is negative, it follows that |-3| = -(-3) = 3. So the expression 3 |-3| is equivalent to 3 • 3 = 3² = 9.

Describe and correct the error in evaluating the expression.

9 + 2 x 3 = 11 x 3 = 33

How would you write this problem with the multiplication first?

Answers

The error in the solution is adding 9 and 2 before the multiplication and the new expression is (2 x 3) +  9

The description of the error?

The equation and the steps are given as

9 + 2 x 3 = 11 x 3 = 33

The above solution is incorrect

This is so because the product has a higher order of precedence

So, it must be evaluated first

So, the correct solution is

9 + 2 x 3 = 9 + 6 = 15

To rewrite the expression such that the product is done first, we introduce the use of brackets

So, we have

9 + (2 x 3)

This gives

(2 x 3) +  9

Hence, the result of the products is (2 x 3) +  9

Read more about product expression at

https://brainly.com/question/10873737

#SPJ1

Select the correct equation and correct answer to solve for X using the triangle below.

Answers

The correct equation to solve for x is 2(2x-15)=2x-8 , and the value of x is 11 .

in the question ,

from the given figure , we can see that RL=LQ   and PK=KQ which means

that L and K are the mid points of sides RQ and PQ respectively .

The Mid Point Theorem states that

If the line segment adjoining the mid points of any of the sides of the triangle then the line segment is said to be parallel to the third  side and is half of the length of  third side .

which means LK parallel to RP and RP/2=LK  ( ... by mid point theorem)

Substituting the values of LK = 2x-15  and RP = 2x-8 .

(2x-8)/2=2x-15

Multiplying both sides by 2

2x-8 = 2(2x-15)

So, the equation to solve for x is 2(2x-15)=2x-8

Simplifying further

2x-8 = 4x-30

4x-2x= 30-8

2x = 22

x = 11

Therefore , the correct equation to solve for x is 2(2x-15)=2x-8 , and the value of x is 11 .

Learn more about Mid Point Theorem here

https://brainly.com/question/19505549

#SPJ1

Help me answer this.

Answers

iven:

here are givren the expression:

[tex](2z-y)^{20}[/tex]

xplanation:

o find the coefficient of the given expression, we need to use the binomial expansion formula:

So,

From the formula of binomial expansion:

[tex](x+y)^n=\sum_{k\mathop{=}0}^n(n,k)x^ky^{n-k}[/tex]

then,

After using the above formula in the given binomial, the expansion will be:

[tex](2z-y)^{20}=y^{20}-40y^{19}z+760y^{18}z^2-9120y^{17}z^3+77520y^{16}z^4-496128y^{15}z^5+...[/tex]

inal answer:

ence , the coefficient of the given term is shown below:

[tex]77520[/tex]

Olivia and Camille went clothes shopping. Camille spent $14 less than three
times what Olivia spent. In total, they spent $54. How much did each girl
spend?

Answers

By solving a system of equations we can see that Olivia spent $14 and Camiller spent $37.

How much did each girl spend?

Let's define two variables:

x = Amount that Olivia spends.y = Amount that Camille spends

We know that " Camille spent $14 less than three times what Olivia spent" so we can write:

y = 3*x - 14

And "In total, they spent $54"

x + y = 54

Then we have a system of equations:

y = 3*x - 14

x + y = 54

Replacing the first equation into the second one, we get:

x + 3x- 14 = 54

4x - 14 = 54

4x = 54 + 14

4x = 68

x = 68/4 = 17

Now that we know the value of x, we can try to find the value of y.

y = 3x - 14 = 3*17 - 14 = 37

So Olivia spent $14 and Camiller spent $37.

Learn more about systems of equations:

https://brainly.com/question/13729904

#SPJ1

Write .24 as a fraction
in lowest terms.

Answers

Answer:

.24 as a fraction is 6/25

Step-by-step explanation:

1) Let convert our decimal into a percentage so we could write it in the not simplified fraction of 0.24:

.24 = 24% = 24/100

2) Now since we have 24/100, we could now simplify:

24/100 = 12/50 = 6/25

We can't simplify anymore.

Answer: 6/25

Step-by-step explanation:

Find the y intercept (if there is one) put your answer as a ordered pair.
(-2,6) (1,6)

Answers

The y-intercept of the line that passes through points (-2,6) and (1,6) in an ordered pair is ( 0,6 ).

What is the y-intercept of the line that passes through the given points?

The formula for equation of line is expressed as;

y = mx + b

Where m is slope and b is y-intercept.

The slope of a line is expressed as change in y over change in x.

Slope m = (y₂-y₁)/(x₂-x₁)

Given the data in the question;

Point A ( -2,6 )

x₁ = -2y₁ = 6

Point B ( 1, 6 )

x₂ = 1y₂ = 6

First, we determine the slope of the line.

Slope m = (y₂-y₁)/(x₂-x₁)

Slope m = ( 6 - 6 )/( 1 - ( -2) )

Slope m = ( 0 )/( 1 + 2) )

Slope m = ( 0 )/3

Slope m = 0

Now, we can determine the y-intercept b, using the slope and coordinates of 1 point.

y = mx + b

6 = (0)(1) + b

Solve for b

6 = 0 + b

b = 6

Therefore, the coordinates of the y-intercept is ( 0,6 ).

Learn more about equation of line here: brainly.com/question/2564656

#SPJ1

Can someone help me with this please and thank you!

Answers

Answer: b

Step-by-step explanation:

Can someone please help me with this geometry question

Answers

Answer A explain it’s A

please answer preferably with steps, but not necessary

Answers

Answer:

Step-by-step explanation:

(5n - 3)/2=n-0.3

Then we solve for n.

Multiply the whole thing by 2.

5n-3=2n-0.6

Add 0.6

5n-2.4=2n

Add 2.4

5n=2n+2.4

Subtract 2n

3n=2.4

Divide by 3

n=0.8

3x-1/5=x+1/3 pls help me

Answers

Answer:

x=4/15

Step-by-step explanation:

first you need to make common denominators for 1/5 and 1/3

the common denominator is 15

5*3=15, 3*5=15. 1*3=3, 1*5=5

new equation:3x-3/15=x+5/15

now subtract x from both sides: 3x-3/15=x+5/15

                                                     -x          -x

                                                     2x-3/15=5/15

now you can add 3/15 to 5/15

2x=8/15

now for the last step, divide by 2 because there are 2 x's

2x/2=x         8/15 /2=4/15

x=4/15

Answer:

4/15

Step-by-step explanation:

3x-1/5 = x+1/3

make a common denominator

3x -3/15 = x + 5/15

next you can take away one x from both sides

2x - 3/15 = 5/15

then we want to have x alone on one side and everything else on the other

2x = 5/15 + 3/15

2x = 8/15

1x = 8/15 ÷ 2

1x = 8/30

simplify that

x = 4/15

Other Questions
Believe it or not, there are people out in the world who believe the Holocaustnever happened. How does the information in this article prove otherwise?Provide details from the text to support your answer. help me im having trouble solving this the ""value"" portion of the customer value proposition is how much you could expect to sell one share of your company in an initial public offering (ipo). Riley works at the Six Flags Amusement Park on weekends and usually operates the Merry-Go-Round ride. If she stands on the ride's platform a distance of 6.63 m from its center and reaches a speed of 3.89 m/s, then what is the net inward force that would be required to keep her 50.1-kg body moving in a circle?Question 20 options:114.3 N2.3 N29.5 N1249.5 N Noah invested $2500 in an account paying 3% simple interest. What was the totalamount in his account after 5 years? an investor purchased 100 shares of acme shoelace stock for $20 per share. four years later, the investor sold the stock for $28 per share. this investor would report these transactions, on a per share basis, as What is the relationship between ask phase of data analysis and plan phase of data analysis help please I rlly need help As someone who works with classified information, what should you do if you are contacted by a foreign national seeking information on a research project?. What is f(g(13))?A mapping diagram is shown. a gerontologic nurse practitioner has a large client population with heart disease problems. this nurse practitioner is aware that heart disease is the leading cause of death in the aging adult. what is the cause of this trend? What is the radical form of the expression? (5x3y4)37 Question 1 5 pts Gavin decides to eat the Unknown Taco mainly because O it is the first time anyone in the family has offered him one O he has had an Unknown Taco before and it wasn't so bad O he is hungry O he loves spicy foods David has an ear infection. His doctor told him to put 3 drops of medicine in each ear daily. His prescription is for 30 mL. If each drop is 0.5 mL, how many days will his prescription last Why do scientists think gases such as CO2 trap heat in the atmosphere? Make x the subject of the formulay/x= abc What are radicals in chemistry? do you think the founding fathers did a good job of answering the question, how does a nation implement principles of popular majority rule while at the same time preserving stable governments that protect the rights and liberties of all citizens, use specifics from text? The Fall has brought about human domination which has often resulted in class systems.TrueFalse Written as a simplified polynomial in standard form, what is the result when (x+7)^2 is subtracted from7